2
$\begingroup$

I recently asked about the minimum number of black (forbidden) squares needed to guarantee a unique "loop" cycle on an $n \times n$ grid. It seems natural, then, to consider $m \times n$ rectangular grid - which might be more insightful.

What is the minimum number of black (forbidden) squares needed on an $m \times n$ grid of squares to guarantee that there is only one possible closed loop visiting every white square exactly once? Denote this minimum number $\mathcal{M}(m, n)$

More details are available on the linked (original) post. Misha Lavrov has given an upper bound that $\mathcal{M}(4k, 4k) \leq 2k$. I do not know of a linear lower bound. Experimental methods are given in the same post, and I have performed my own calculations (my code is very crude and uses exhaustive search; if I were a better coder, I would probably restrict the search space). Below are the values $\mathcal{M}(m,n)$ I have computed. (Rob Pratt has given more values, which I have added)

\begin{array}{c|ccccccccccccc} m \backslash n & 2 & 3 & 4 & 5 & 6 & 7 & 8 & 9 & 10 & 11 & 12 & 13 & 14 \\ \hline 2 & 0 & 0 & 0 & 0 & 0 & 0 & 0 & 0 & 0 & 0 & 0 & 0 & \dots \\ 3 & 0 & 1 & 2 & 1 & 2 & 3 & 2 & 3 & 4 & 3 & 4 & \dots & \dots \\ 4 & 0 & 2 & 2 & 2 & 2 & 2 & 2 & 2 & 2 & \dots & \dots & \dots & \dots \\ 5 & 0 & 1 & 2 & 3 & 4 & 3& 4 & 5 & 4 & \dots & \dots & \dots & \dots \\ 6 & 0 & 2 & 2 & 4 & 4 & 4 & 4 & 4 & \dots & \dots & \dots & \dots & \dots \\ 7 & 0 & 3 & 2 & 3 & 4 & 5 & 4 & \dots & \dots & \dots & \dots & \dots & \dots \\ 8 & 0 & 2 & 2 & 4 & 4 & 4 & 4 & 4 & \dots & \dots & \dots & \dots & \dots \\ 9 & 0 & 3 & 2 & 5 & 4 & \dots & 4 & \dots & \dots & \dots & \dots & \dots & \dots \\ 10 & 0 & 4 & 2 & 4 & \dots & \dots & \dots & \dots & \dots & \dots & \dots & \dots & \dots \\ 11 & 0 & 3 & \dots & \dots & \dots & \dots & \dots & \dots & \dots & \dots & \dots & \dots & \dots \\ 12 & 0 & 4 & \dots & \dots & \dots & \dots & \dots & \dots & \dots & \dots & \dots & \dots & \dots \\ \end{array}

The table is far from complete. Of course, $\mathcal{M}(m,n) = \mathcal{M}(n,m)$ and $\mathcal{M}(2,n) = 0$. For larger values, the problem is more interesting. I have already found patterns that the 'optimal' arrangement of black squares must follow for $\mathcal{M}(3,n)$, namely

$$\mathcal{M}(3,n) = \left\lfloor \frac{n}{3} \right\rfloor + \begin{cases} 1 & \text{if } n \equiv 1 \pmod 3,\\[2mm] 0 & \text{otherwise.} \end{cases}$$

For $\mathcal{M}(4,n)$, I am stumped. The long sequence of $2,2,2,2, \dots$ makes me think $\mathcal{M}(4,n) = 2$ for $n > 2$, but I do not want to mistakenly extrapolate anything. I will continue to review the arrangements for $\mathcal{M}(4,n)$; perhaps I have missed something obvious. After $4$, it becomes increasingly (exponentially...) hard to fill the table.


I am looking for any insight into $\mathcal{M}(m,n)$. This includes a linear lower bound, the calculation of values, etc.

$\endgroup$
1
  • $\begingroup$ The $\mathcal M(4k, 4k)$ construction immediately extends to $\mathcal M(4k,n)$ for any even $n\ge 4$ with no changes. For odd $n$, I believe it's enough to simply take the mirror image of the next-to-last row, but I haven't carefully checked. (By the way, the construction is generally very flexible about how you "tie it off" at the end, so it is far from unique.) So I think $\mathcal M(4k, n) \le 2k$ for all $n$, and since $\mathcal M(4,n)$ is even and positive for $n>2$, this would confirm $\mathcal M(4,n) = 2$ for all $n>2$. $\endgroup$ Commented Nov 17 at 0:57

1 Answer 1

3
$\begingroup$

I confirm your computed values, and here are some more: \begin{matrix} \hline m & n & \mathcal{M}(m,n) \\ \hline 5 & 7 & 3 \\ 5 & 8 & 4 \\ 5 & 9 & 5 \\ 5 & 10 & 4 \\ 6 & 7 & 4 \\ 6 & 8 & 4 \\ 6 & 9 & 4 \\ 7 & 8 & 4 \\ 8 & 9 & 4 \\ \hline \end{matrix}

$\endgroup$

You must log in to answer this question.

Start asking to get answers

Find the answer to your question by asking.

Ask question

Explore related questions

See similar questions with these tags.